Altitudes AD and BE of acute triangle ABC intersect at point H. If angle AHB = 123 degrees and angle BAH = 26 degrees, then what is angle HCA in degrees?

Asymptote code below
[asy]
size(150); defaultpen(linewidth(0.8));
pair B = (0,0), C = (3,0), A = (1.1,2), P = foot(A,B,C), Q = foot(B,A,C),H = intersectionpoint(B--Q,A--P);
draw(A--B--C--cycle);
draw(A--P^^B--Q);
label("$A$",A,N); label("$B$",B,W); label("$C$",C,E); label("$D$",P,S); label("$E$",Q,NE); label("$H$",H,NW);
[/asy]

Please, there's a message board for a reason

To find angle HCA, we can use the fact that the sum of the angles in a triangle is 180 degrees.

Since AD is an altitude, angle BAH is a right angle. Therefore, angle BAC + angle CAH = 90 degrees.

Similarly, since BE is an altitude, angle AHB is a right angle. Therefore, angle ABC + angle CBH = 90 degrees.

We are given that angle BAH = 26 degrees and angle AHB = 123 degrees.

To find angle HCA, we can subtract the sum of angle BAH and angle ABC from 90 degrees:

angle HCA = 90 degrees - (angle BAH + angle ABC)

angle HCA = 90 degrees - (26 degrees + angle ABC)

Now, we need to find angle ABC. In triangle ABC, angle BCA can be found by subtracting the sum of angle BAC and angle ACB from 180 degrees:

angle BCA = 180 degrees - (angle BAC + angle ACB)

Since angle ACB + angle CAH = 90 degrees (as discussed earlier), we can substitute this value in:

angle BCA = 180 degrees - (angle BAC + 90 degrees - angle BCA)

Simplifying,

angle BCA = 180 degrees - angle BAC - 90 degrees + angle BCA

angle BCA = 90 degrees - angle BAC + angle BCA

Now, we can substitute the given values:

angle BCA = 90 degrees - 26 degrees + angle BCA

angle BCA = 64 degrees + angle BCA

Subtracting angle BCA from both sides,

0 = 64 degrees

This means that angle BCA is 0 degrees, which is not possible in a triangle.

Therefore, there is no solution for the given conditions.

AOPS PROE.

31

The altitudes of a triangle intersect at the orthocenter of the triangle. So, when we extend $\overline{CH}$ to point $F$ on $\overline{AB}$, we have $\overline{CF}\perp\overline{AB}$ as shown below.

[asy]
size(150); defaultpen(linewidth(0.8));
pair B = (0,0), C = (3,0), A = (1.1,2), P = foot(A,B,C), Q = foot(B,A,C),H = intersectionpoint(B--Q,A--P);
draw(A--B--C--cycle);
draw(A--P^^B--Q);
label("$A$",A,N); label("$B$",B,W); label("$C$",C,E); label("$D$",P,S); label("$E$",Q,NE); label("$H$",H+(0,-0.23),SW);
pair F = foot(C,A,B);
draw(C--F);
label("$F$",F,NW);
draw(rightanglemark(C,P,H,3.5));
draw(rightanglemark(H,F,A,3.5));
draw(rightanglemark(H,Q,C,3.5));
[/asy]

From $\triangle HBA$, we have $\angle HBA = 180^\circ - 123^\circ - 26^\circ = 31^\circ$, and from right triangle $BEA$, we have
\[\angle HBA = \angle ABE = 90^\circ - \angle BAE=90^\circ - \angle BAC.\]
But from right triangle $CFA$, we have
\[\angle HCA = \angle FCA = 90^\circ - \angle FAC = 90^\circ - \angle BAC,\]which matches our expression for $\angle HBA$. So, we have $\angle HCA = \angle HBA = \boxed{31^\circ}$.

We would appreciate it if you worked out the problems by yourself instead of having another person do it for you. If you have a question, you may use the message board to ask us but it is best if you do not use the internet.

Have fun solving math problems, and you can always ask for help!
🙂

nooooooooooooooooooooooooooooooooooooo

First, I'm sorry but there is a message board for a reason. Please use it, students.Secondly, this answer is wrong.

Angle HCA is 26 degrees.